1% defective parts. 100,00 parts made in total. The number of defects made should equal?

Answers

Answer 1

Answer:

1,000 defects

Step-by-step explanation:

Find how many defects that should be made by finding 1% of 100,000:

100,000(0.01)

= 1000

So, there should be 1,000 defects


Related Questions

log13 X + log13 (12x-1)=1

Answers

Solve for  x  by simplifying both sides of the equation, then isolating the variable.

x  ≈  0.13893498

A reacher deposited Rs 66000 in his/her saving account for as many years as the rate of interest per annum .If he/ she received Rs 10560 as interest at the end of saving period find the time duration and rate of interest.
Help me pls
I need the answer as quickly as possible ​

Answers

Answer:

Time and rate is 4 years and 4% respectively.

Step-by-step explanation:

P = Rs 66000

so

T = R

I = 10560

so

I = PTR/100

Rs 10560 = (66000*T^2)/100

or, Rs 1056000 = 66000*T^2

or. 1056000/66000 = T^2

or, 16 = T^2

OR, T =√16

so T = 4

Then

T = R

so, R = 4

How many 1.5 ft
pieces of pipe can you cut from a 12ft pipe

Answers

Answer:

8 pieces.

Step-by-step explanation:

12 / 1.5

= 24 / 3

= 8.

Lấy ngẫu nhiên 25 lọ thuốc Vitamin tổng hợp do một máy tự động đóng chai ta thu được s'2 =0,012(l2).Máyđượcgọilàđạtchuẩnnếuđộphântántheoquyđịnhlà2 =0,005(l2). Với mức ý nghĩa 0,05 hãy kiểm định máy đóng chai có đạt chuẩn không? Biết lượng thuốc trong chai là biến ngẫu nhiên có phân phối chuẩn.

Answers

Answer:

I really can't read this

Step-by-step explanation:

Which of the following algebraic steps will solve the equation 5p= -35 and what is the solution

Answers

Answer:

p=-7

Step-by-step explanation:

5p=-35

Divide both sides by 5

5p/5 =-35/5

therefore p=-7

[tex]\huge\text{Hey there!}[/tex]

[tex]\large\textsf{5p= -35}\\\\\underline{\huge\text{DIVIDE 5 BOTH SIDES}}\\\\\mathsf{\dfrac{5p}{5p}= \dfrac{-35}{5}}\\\\\large\text{CANCEL out: }\dfrac{5}{5}\large\text{ because that gives you 1}\\\large\text{KEEP: }\dfrac{-35}{5}\large\text{ because that gives you the value of p.}\\\\\large\textsf{p = }\mathsf{\dfrac{-35}{5}}\\\\\mathsf{-35\div5= p}\\\\\large\text{Simplify above and you have your result to the p-value.}\\\\\boxed{\boxed{\huge\text{Therefore your answer is: \textsf{p = -7}}}}\huge\checkmark[/tex]

[tex]\huge\text{Good luck on your assignment \& enjoy your day!}[/tex]

~[tex]\frak{Amphitrite1040:)}[/tex]

Find f(2) given f(x) = -3x^3 + x^2 – 3

Answers

Answer:

D

Step-by-step explanation:

f(x) = -3x^3 + x^2 – 3       f(2) means that wherever you see a x, put in a 2.

f(2)= -3(2)^3 + (2)^2 - 3

f(2) = -3*8 + 4 - 3

f(2) = - 24 + 1

f(2) = - 23

f(x)=-3x^3+x^2-3

[tex]\\ \sf \longmapsto f(2)[/tex]

[tex]\\ \sf \longmapsto -3(2)^3+(2)^2-3[/tex]

[tex]\\ \sf \longmapsto -3(8)+4-3[/tex]

[tex]\\ \sf \longmapsto -24+1[/tex]

[tex]\\ \sf \longmapsto -23[/tex]

Can someone help me find the answer?

Answers

Answer:

a. x = 3/a

Step-by-step explanation:

Add all like terms on left hand side of the equation:

5 ax + 3 ax => 8 ax

Bring like term 4ax on left hand side

8ax - 4ax

=> 4ax

Therefore we get 4ax = 12

ax = 12/4

ax = 3

x = 3/a

Which equation represents the data in
the table?
x 0 1 2 3 4
y -4 -2 0 2 4
F y= x -4
G y= 2x -2
H y = 2x - 4
I y= 4x -4

Answers

Answer:

y = 2x - 4

Step-by-step explanation:

The equations are put in slope intercept form

Slope intercept form: y = mx + b

Where m = slope and b = y intersect

So in order to find the equation of the data represented by the table we will have to find the slope and y intercept

Let's begin!

First let's find the slope

We can find the slope by using the slope formula

m = (y2 - y1) / (x2 - x1) where the x and y values are derived from coordinates from the table

The points chosen may vary but I have chosen the points (0,-4) and (1,-2)

Now that we have chosen the points we will use to find the slope let's define the variables

remember coordinates are written like this: (x,y)

The x value of the second coordinate is 1 so x2 = 1

The x value of the first coordinate is 0

So x1 = 0

The y value of the second coordinate is -2 so y2 = -2

The y value of the first coordinate is -4

So y1 = -4

Now that we have defined each variable let's plug in the values into the formula

Formula: m = (y2 - y1) / (x2 - x1)

Variables: x2 = 1, x1 = 0, y2 = -2, y1 = -4

Substitute values

m = (-2 - (-4) / ( 1 - 0 )

Evaluate

The negative signs cancel out on top and it changes to +4

m = (-2 + 4)/(1-0)

Add top values

m = 2/(1-0)

Subtract bottom numbers

m = 2/1

Simplify fraction

m = 2

So we can conclude that the slope (m) = 2

Now let's find the y intercept or "b"

The y intercept is the value of y when x = 0

If you look at the table when x = 0 y = -4 meaning that the y intercept or "b" is -4

Now that we have found everything let's find the equation of the data represented by the table

The equation is in slope intercept form

y = mx + b

Define variables

m = 2 and b = -4

Substitute values

y = 2x - 4

The equation is y = 2x - 4

PLS HELP
Let f(x) = -2x - 7 and g(x) = -4x + 6. Find (g o f) (-5)


–6



3



–59



26

Answers

Answer:

-6

Step-by-step explanation:

f(x) = -2x - 7 and g(x) = -4x + 6

Find f(-5)

f(-5) = -2(-5) -7 = 10-7 = 3

The find g(3)

g(3) = -4(3) +6

     = -12 +6 =-6

g(f(-5)) = -6

A boat travels 400 kilometers in 9.6 hours (with a constant speed). How much time will it take to travel 138 kilometers? (round to the nearest tenth of an hour)

Answers

Step-by-step explanation:

here's the answer to your question


constant of proportionality

Answers

Answer:

4.2

Step-by-step explanation:

Constant of proportionality = y/x = 8.4 / 2 = 4.2

You are skiing down a mountain with a vertical height of 1250 feet. The distance that you ski as you go from the top down to the base of the mountain is 3350 feet. Find the angle of elevation from the base to the tep of the mountain. Round your answer to a whole number as necessary. ​

Answers

Step-by-step explanation:

here is the answer to your question

F(x, y, z) = (x + yz)i + (y + xz)j + (z + xy)k, Find the divergence of the vector field.

Answers

The divergence of F is

div(F ) = ∂(x + yz)/∂x + ∂(y + xz)/∂y + ∂(z + xy)/∂z

div(F ) = 1 + 1 + 1

div(F ) = 3

The divergence of the vector field is equal to 3

Data;

(x+yz)i(y + xz)j(z+xy)kDivergence of Vector Field

To find the divergence of the vector field, we have to differentiate the i, j and k component of the vector.

[tex]div F = \frac{\delta}{\delta x} (x+yz) + \frac{\delta}{\delta y} (y + xz) + \frac{\delta}{\delta z} (z + xy)\\div F = (1+0)+(1+0)+(1+0)\\div F = 1 + 1 + 1 \\div F = 3[/tex]

The divergence of the vector field is equal to 3

Learn more on divrgence of vector field here;

https://brainly.com/question/4608972

Company A charges a $125 annual fee plus $7 per hour car share fee.
Company B charges $110 plus $10 per hour. What is the minimum number of
hours that a car share needs to be used per year to make company A a better
deal?
A. 6
O Ο Ο
B. 5
C. 9
D. 11
SUBMIT
PREVIOUS

Answers

Answer:

6 hours

Step-by-step explanation:

A: y=125+7x

B: y=110+10x

for A to be a better deal than B, 125+7x<110+10x has to be true

subtract 7x from both sides and subtract 110 from both sides of the inequality: 15<3xdivide by 3: 5<xso x has to be greater than 5, im pretty sure it's 6 hours but it might also be 5 depending on how it's taught for you

Answer:

B 5

Step-by-step explanation:

Company A

total cost = 125+7h  where h is the number of hours

Company B

total cost = 110 + 10h  where h is the number of hours

125 + 7h < 110 + 10h

Subtract 7 h from each side

125 + 7h-7h<110+10h-7h

125 < 110+3h

Subtract 110 from each side

125-110< 110+3h

15 <3h

Divide by 3

15/3 <3h/3

5 <h

More than 5 hours


Find the face value of the 20-year zero-coupon bond at 4.4%, compounded semiannually, with a price of $8,375.
$45.000
$53.000

Answers

The correct face value will be Option C ($20,000). A further solution id provided below.

Given:

Time,

t = 20 years

Rate,

r = 4.4%

Price

= $8,375

Now,

The yield will be:

= [tex]\frac{4.4}{2}[/tex]

= [tex]1.1[/tex] (%)

Time will be:

= [tex]20\times 2[/tex]

= [tex]40 \ periods[/tex]

As we know the formula,

⇒ [tex]Price \ of \ bond = \frac{Face \ value}{(1+\frac{r}{2} )^{n\times 2}}[/tex]

By substituting the values, we get

                   [tex]8375=\frac{Face \ value}{(1+\frac{0.044}{2} )^{20\times 2}}[/tex]

                   [tex]8375=\frac{Face \ value}{(1.022)^{40}}[/tex]

                   [tex]8375=\frac{Face \ value}{2.3880083}[/tex]

The face value will be:

        [tex]Face \ value = 2.3880083\times 8375[/tex]

                          [tex]=20,000[/tex] ($)

Learn more about face value here:

https://brainly.com/question/14862802

Question
If a triangle has sides of length x x + 2, and x - 4, what is the perimeter of the triangle in terms of x?
О 3х - 6
03x - 2
3x + 2
O 3x + 6

Answers

9514 1404 393

Answer:

  (b)  3x -2

Step-by-step explanation:

The perimeter is the sum of the side lengths:

  P = (x) +(x +2) +(x -4)

  P = (x +x +x) +(+2 -4)

  P = 3x -2

WILL GIVE BRAINLIEST

Use the distributive property.
5(2x + 7) = [ ? ]x + [ ]

Answers

Answer:

10x +35

Step-by-step explanation:

5(2x+7) = 5*2x + 5*7 = 10x+35

Answer:

10, 35.

Step-by-step explanation:

5(2x + 7)

= 5^2x + 5*7

= 10x + 35

Solve the following equation for x. 12^2 - 36x = 0

Answers

The answer is that X is equal to 4

An operation manager at an electronics company wants to test their amplifiers. The design engineer claims they have a mean output of 331331 watts with a variance of 144144. What is the probability that the mean amplifier output would be greater than 332.8332.8 watts in a sample of 4949 amplifiers if the claim is true

Answers

Answer: brother

Step-by-step explanation:

The sum of 'n' terms of an arithmetic sequence is 4n2+3n. What is the first term, the common difference, and the sequence?

Answers

Answer:

Step-by-step explanation:

If the 2 is an exponent, it should be indicated with a circumflex:  4n^2+3n.

What is the first term?

a₁ = S₁ = 4·1² + 3·1 = 7

:::::

What is the common difference?

a₁ + a₂ = S₂ = 4·2² + 3·2 = 22

a₂ = 22-a₁ = 15

common difference d = a₂ - a₁ = 8

:::::

What is the arithmetic sequence?

The nth term is 7+(n-1)d = 7+(n-1)8 = 8n-1.

a₁ = 8·1-1 = 7

a₂ = 8·2-1 = 15

a₃ = 8·3-1 = 23

...

Given the numbers 30 and 45, find the common factors of the two numbers.​​

Answers

Factors of 30: 1, 2, 3, 5, 6, 10, 15, 30

Factors of 45: 1, 3, 5, 9, 15, 45

The common factors between the two numbers are 1, 3, 5, 15.

Hope this helps!

What is the slope-intercept equation of the line below?

Answers

Answer:

y=-5/4x+3

Step-by-step explanation:

y=mx+c

m = (y1-y2)/(x1-x2) =(-2-3)/(4-0) =-5/4

sub the values y=3, x=0 to find c,

3 =-5/4(0) + c

c = 3

Someone help me please

Answers

Answer:

y=4

Step-by-step explanation:

The triangle on the right is a 30-60-90 triangle

The ratio of sides in a 30-60-90 triangle is 1:√3:2

y:x:8

SO y will be 4 because 8/y=2/1

Do the two trapezoids in the figure appear to be similar? Why or why not?
options:

A)

They're not similar because only one pair of corresponding angles in the two trapezoids is congruent.

B)

They're similar because two pairs of corresponding angles in the two trapezoids are congruent.

C)

They're similar because one pair of corresponding angles in the two trapezoids is congruent.

D)

They're not similar because two pairs of corresponding angles in the two trapezoids are congruent.

Answers

Answer:

Option B,

They're similar because two pairs of corresponding angles in the two trapezoids are congruent

Answer:

They're not similar because only one pair of corresponding angles in the two trapezoids is congruent.

Step-by-step explanation:

HELP ME PLSSS SUMMER SCHOOL A HARD

Answers

Answer:

y=2x+8

Step-by-step explanation:

Hope this helps

Can you help with this

Answers

9514 1404 393

Answer:

  D, C

Step-by-step explanation:

The only two rational expressions that have appropriate denominators are ...

  1/(x² +6x) . . . contributes a factor of x to the denominator

  (x+2)/(x² -36) . . . contributes a factor of (x -6) to the denominator

The proper order in the expression is ...

  [tex]\displaystyle\dfrac{x+2}{x^2-36}-\dfrac{1}{x^2+6x}=\dfrac{x^2+x+6}{x(x-6)(x+6)}[/tex]

Answer:

(x+2/x^2-36) - (1/x^2+6x) = (x^2+x+6/x(x-6)(x+6))

Step-by-step explanation:

I hope that helps

What is the greatest common factor of 10 and 13?
ANSWER IN LESS THEN 3 MINS AND I'LL GIVE BRAINLIEST

Answers

Answer:

the answer for this would be 1

. Two mutually exclusive projects have projected cash flows as follows:
YEAR PROJECT A PROJECT B
0 Ksh. -2m Ksh. -2m
1 1m 0
2 1m 0
3 1m 0
4 1m 6m





Required:
a) Determine the internal rate of return for each project. [2 Marks]

b) Determine the net present value for each project at discount rates of 0, 5,10,20,30, and 35 percent. [2 Marks]

c) Plot a graph of the net present value of each project at the different discount rates. [2 Marks]

d) Which project would you choose? Why? [ 2 Marks]

e) What is each project’s MIRR if the cost of capital is 12 percent?

Answers

Answer:

yes

Step-by-step explanation:

 In one of the examples he is working on, he knows that the two coordinates (0,6) and
(8, 10) are on the function that he is deriving. Using the information from these two
coordinates, determine the slope and y-intercept of the function Mike is looking for, and
then write out the correct function.

Answers

9514 1404 393

Answer:

  y = 1/2x + 6

Step-by-step explanation:

The slope m is given by the formula ...

  m = (y2 -y1)/(x2 -x1)

  m = (10 -6)/(8 -0) = 4/8 = 1/2

The y-intercept is given by the formula ...

  b = y -mx

  b = 6 -(1/2)(0) = 6

Then the slope-intercept equation is ...

  y = mx +b

  y = 1/2x +6

show that the line x-y+2=0 and the line joining the points (4,6) and (10,12) are parallel to each other

Answers

x-y+2=0

-y= -x-2

y=x+2

In order for the other line to be parallel, the slope has to be the same.

(12-6)/(10-4)= 6/6= 1

Both slopes are the same, meaning it is parallel.

Other Questions
PLS HELP ME ON THIS QUESTION I WILL MARK YOU AS BRAINLIEST IF YOU KNOW THE ANSWER PLS GIVE ME A STEP BY STEP EXPLANATION!!Data should be analyzed using each of the following except:A. measures of central tendencyB. shapeC. population sizeD. spread Use the quadratic formula to find the solutions to the quadratic equation below. Check all that apply. 5x2-X-4 = 0 A. -4/5 B. 5/4C. 2/3 D. 1 E. -1 F.3/2 a(b^2+c^2+bc) + b(c^2+a^2+ca) + c(a^2+b^2+ab) The element of an argument that ties everything together is the __________.evidencereasoningwarrantclaim Can you please help me with this question TotalWinnings0Translate You are going to study into Spanish in the box below: What is a drawing called that is made when figuring out where things will go in a final composition? A basic B loose C preliminary D formal write an essay on clean environment If 7 4 = 28, what value should be subtracted from 28 to find the product of 6 4?2346 Define trade deficit The management team at Electronics Galaxy is evaluating whether to have sales staff wear uniforms on the showroom floor. What should they consider about wearing uniforms ? a) How staff are dressed doesn't usually have much of an impact on customers. b) Uniforms can help customers identify members of the sales staff. Team morale usually suffers when uniforms are put in place . c) d) Uniforms can take away from each employee's individual personality. In what two cities was the atomic bomb dropped in Japan?A. Nagasaki and TokyoB. Hiroshima and TokyoC. Hiroshima and NagasakiD. Tokyo and Seoul a word problem on proportions using a unit rateLashonda made $273 for 13 hours of work.At the same rate, how many hours would she have to work to make $231?hours? Daphne Inc., a steel manufacturing company, is planning to buy a new plant at $1,090,000. The life of the plant is estimated to be 5 years and has cash flows of $109,000, $218,000, $327,000, $436,000, and $545,000. Calculate the payback period for the new plant. a. 5 years b. 2 years c. 4 years d. 3 years Who were the first settlers in Virginia steel wire 8m long and 4mm in diameter is fixed to two rigid b. [1] supports. Calculate the increase in tension when the temperature falls by 10C. Given linear expansivity of steel, a 12x10 %/K, Young's modulus for steel, Y = 2x10 N/m. Question 1 of 10Is (0,0) a solution to this system?A. No. (0,0) does not satisfy either inequality.B. No. (0,0) satisfies y< x2 + 2x + 1 but does not satisfy yz x2 + x - 4.C. No. (0,0) satisfies ya x2 + x - 4 but does not satisfy y< x2 + 2x + 1.OD. Yes. (0,0) satisfies both inequalities. Put the following equation of a line into slope-intercept form, simplifying allfractions.3x + 6y = -42 Quiero que comas algo ligero esta noche, as que no tomes comida procesada o bebas batidos.Qu verbo en subjuntivo se usa en una expresin de influencia?tomes ,Quiero , ,comas , ,bebas What is the Square root of 30 ,12,36